K
Khách

Hãy nhập câu hỏi của bạn vào đây, nếu là tài khoản VIP, bạn sẽ được ưu tiên trả lời.

27 tháng 9 2017

Giả sử tồn tại số tự nhiên n sao cho \(n^2+5n-13⋮121\)

\(\Leftrightarrow\left(n^2-6n+9\right)+11n-22⋮11\) ( Do \(121⋮11\) )

\(\Leftrightarrow\left(n-3\right)^2+11\left(n-2\right)⋮11\)

\(\Rightarrow\left(n-3\right)^2⋮11\)

Mà 11 là số nguyên tố \(\Rightarrow n-3⋮11\) \(\Rightarrow n=11a+3\left(a\in N\right)\)Thay n = 11a + 3 vào ta có:\(\left(11a+3\right)^2+5\left(11a+3\right)-13=121a^2+121a+11⋮̸121\)

\(\Rightarrow\) Vô lí điều ta đã giả sử

\(\Rightarrow\) \(\forall n\in N\) thì \(n^2+5n-13⋮̸121\) ( đpcm)

1 tháng 5 2015

ta có: n2+n+1= (n+2)(n-1) +3 
ta thấy hiệu hai số: (n+2) -(n-1) =3 chia hết cho 3 
suy ra: 
( *) hoặc (n+2) và (n-1) cùng chia hết cho 3, khi đó (n+2)(n-1) chia hết cho 9 nhưng 3 không chia hết cho 9 , dó đó (n+2)(n-1) +3 không chia hết cho 9 hay n2+n+1 không chia hết cho 9 
(**) hoặc (n+2) và (n-1) cùng không chia hết cho 3, khi đó (n+2)(n-1) ko chia hết cho 3,suy ra (n+2)(n-1) +3 ko chia hết cho 3. Mà đã không chia hết cho 3 thì đương nhiên không chia hết cho 9 rồi
------Cho 1 Đ.ú.n,g nhé

16 tháng 9 2023

Đặt n = 3k \(\left(k\inℕ\right)\)

Khi đó P = 9k2 + 3k + 1 = 3k(3k + 1) + 1 \(⋮̸3\)

=> \(P⋮̸9\)

Tương tự với n = 3k + 1

P = 9k2 + 9k + 3 = 9k(k + 1) + 3\(⋮̸9\)

Với n = 3k + 2 

P = 9k2 + 15k + 7 = 3k(3k + 5) + 7 \(⋮̸3\Leftrightarrow P⋮̸9\)

=> ĐPCM 

17 tháng 9 2018

Xet \(n=3k\)

\(\left(3k\right)^2+3k+2\equiv2\left(mod3\right)\)

Xet \(n=3k+1\)

\(\left(3k+1\right)^2+3k+1+2\equiv4\equiv1\left(mod3\right)\)

Xet \(n=3k+2\)

\(\left(3k+2\right)^2+3k+2+2\equiv1+2+2\equiv2\left(mod3\right)\)

\(\Rightarrow n^2+n+2⋮̸3\)

\(\Rightarrow n^2+n+2⋮̸15\)

17 tháng 9 2018

Mod là sao

29 tháng 7 2017

Xét \(n=2k+1\)

\(\Rightarrow A=3^{2k+1}+1=3.9^k+1\)

Ta có: \(9^k\) chia cho 5 dư - 1 hoặc 1 

\(\Rightarrow3.9^k\)chia 5 dư - 3 hoặc 3

\(\Rightarrow3.9^k+1\)chia 5 dư - 2  hoặc 4

\(\Rightarrow A\) không chia hết cho 5 nên A không chia hết cho \(10^{2016}\)

Xét \(n=2k\)

\(\Rightarrow A=3^{2k}+1=3^{2k}+1\)

Vì \(3^{2k}\)là số chính phương nên chia cho 4 dư 0 hoặc 1.

\(\Rightarrow A=3^{2k}+1\)chia cho 4 dư 1 hoặc 2.

\(\Rightarrow A\)không chia hết cho 4 nên A không chia hết cho \(10^{2016}\)

6 tháng 3 2018

Ta có: \(E=36^n+19^n-2^n\cdot2\)

Mặt khác: \(36\equiv19\equiv2\)(mod 17)

Do đó: \(VT\equiv2^n+2^n-2^n\cdot2\equiv0\)(mod 17)

Vậy .................